Difference between revisions of "2006 AMC 10A Problems/Problem 1"

m (added category and link to next problem)
 
(6 intermediate revisions by 5 users not shown)
Line 1: Line 1:
== Problem ==
+
#redirect [[2006 AMC 12A Problems/Problem 1]]
Sandwiches at Joe's Fast Food cost <math>3 each and sodas cost </math>2 each.  How many dollars will it cost to purchase 5 sandwiches and 8 sodas?
 
 
 
<math> \mathrm{(A) \ } 31\qquad \mathrm{(B) \ } 32\qquad \mathrm{(C) \ } 33\qquad \mathrm{(D) \ } 34\qquad \mathrm{(E) \ } 35 </math>
 
== Solution ==
 
Our answer is simply
 
<math>(3\cdot 5)+(2\cdot 8)\Longrightarrow 15+16=31,(A)</math>
 
 
 
== See Also ==
 
*[[2006 AMC 10A Problems]]
 
 
 
*[[2006 AMC 10B Problems/Problem 2|Next Problem]]
 
 
 
[[Category:Introductory Algebra Problems]]
 

Latest revision as of 00:00, 28 April 2008